Difference between revisions of "2010 AMC 10B Problems/Problem 24"

(Redirected page to 2010 AMC 12B Problems/Problem 19)
(Tag: New redirect)
 
(45 intermediate revisions by 22 users not shown)
Line 1: Line 1:
Represent the teams' scores as: <math>(a, an, an^2, an^3)</math> and <math>(a, a+m, a+2m, a+3m)</math>
+
#redirect [[2010 AMC 12B Problems/Problem 19]]
 
 
We have <math>a+an+an^2+an^3=4a+6m+1</math>
 
Manipulating this, we can get <math>a(1+n+n^2+n^3)=4a+6m+1</math>, or <math>a(n^4-1)/(n-1)=4a+6m+1</math>
 
 
 
Since both are increasing sequences, <math>n>1</math>. We can check cases up to <math>n=4</math> because when <math>n=5</math>, we get <math>156a>100</math>. When
 
          n=2, a=[1,6]
 
          n=3, a=[1,2]
 
          n=4, a=1
 
Checking each of these cases individually back into the equation a+an+an^2+an^3=4a+6m+1, we see that only when a=5 and n=2, we get an integer value for m, which is 9. The original question asks for the first half scores summed, so we must find (a)+(an)+(a)+(a+m)=(5)+(10)+(5)+(5+9)=34
 

Latest revision as of 20:54, 26 May 2020